Srednicki Problem 3.4: Deriving the Klein-Gordon Equation

  • Thread starter WannabeNewton
  • Start date
  • Tags
    Srednicki
P} = \int d\tilde{k} [a^{\dagger}(\mathbf{k})a(\mathbf{k}) +a(\mathbf{k})a^{\dagger}(\mathbf{k})]\mathbf{k}##.In summary, by recalling the space-time translation operator ##T(a)## and the 4-momentum operator ##P^{\mu}##, we can derive an expression for the commutator ##[P^{\mu},\varphi]## by considering infinitesimal translations. This result is equivalent to the Heisenberg equation of motion for the time component, and can be used to
  • #1
WannabeNewton
Science Advisor
5,844
551

Homework Statement


Recall that ##T(a)^{-1}\varphi(x)T(a) = \varphi(x - a)## where ##T(a) = e^{-iP^{\mu}a_{\mu}}## is the space-time translation operator and ##P^{\mu}## is the 4-momentum operator.

(a) Let ##a^{\mu}## be infinitesimal and derive an expression for ##[P^{\mu},\varphi]##.
(b) Show that the time component of your result is equivalent to the Heisenberg equation of motion ##\dot{\varphi} = i[H,\varphi]##.
(c) For a free field, use the Heisenberg equation to derive the Klein-Gordon equation.
(d) Define a spatial momentum operator ##\mathbf{P} = -\int d^{3}x (\pi \nabla\varphi)##. Use the canonical commutation relations to show that ##\mathbf{P}## obeys the relation you derived in part (a).
(e) Express ##\mathbf{P}## in terms of ##a(\mathbf{k})## and ##a^{\dagger}(\mathbf{k})##.

Homework Equations



The Hamiltonian density is given by ##\mathcal{H} = \frac{1}{2}\pi^2 + \frac{1}{2}(\nabla \varphi)^2 + \frac{1}{2}m^2 \varphi^2## where ##\pi## is the field conjugate momentum as usual.

The equal time commutation relations for the field and the field conjugate momentum are ##[\varphi(x,t),\varphi(x',t)] = 0##, ##[\pi(x,t),\pi(x',t)] = 0##, and ##[\varphi(x,t),\pi(x',t)] = i\delta^{3}(\mathbf{x} - \mathbf{x'})##.

You probably already noticed but ##\varphi## is a real field. Also I'll use the notation ##\varphi' \equiv \varphi(x',t)## with ##\pi'## and ##\nabla'## defined similarly just to make things cleaner.

The Attempt at a Solution



(a) ##\varphi(x^{\mu} - a^{\mu}) = \varphi(x^{\mu}) - a_{\mu}\partial^{\mu}\varphi +O(a^2)## and ##T(a^{\mu}) = I - ia_{\mu}P^{\mu}+O(a^2)## so ##T^{-1}\varphi T = (I + ia_{\mu}P^{\mu})(\varphi - ia_{\mu}\varphi P^{\mu}) = \varphi(x^{\mu}) + ia_{\mu}[P^{\mu},\varphi]+O(a^2)## hence ##i[P^{\mu},\varphi] = -\partial^{\mu}\varphi## since ##a_{\mu}## was arbitrary.

(b) ##\mu = 0## gives ##i[H,\varphi] = -\partial^{0}\varphi = \dot{\varphi}##

(c) We have ##2[\varphi,H] = [\varphi,\int d^3{x'}(\pi'^2 + (\nabla'\varphi')^2 + m^2 \varphi'^2)] = \int d^{3}x'([\varphi,\pi'^2] + [\varphi,(\nabla'\varphi')^2] + m^2[\varphi,\varphi'^2])##; the equal time commutator can be pulled into the integral since the commutator is evaluated at the field point ##x## whereas the integral is over ##x'##.

Now ##[\varphi,\nabla'\varphi'] = \nabla'[\varphi,\varphi'] = 0## hence ##[\varphi,(\nabla'\varphi')^2] = 0##. Similarly ##[\varphi,\varphi'^2] = 0##. This leaves us with ##2[\varphi,H] = \int d^{3}x'[\varphi,\pi'^2] ##.

We have ##[\varphi,\pi'^2]= i\delta^{3}(\mathbf{x} - \mathbf{x}')\pi' + (\pi' \varphi)\pi' - (\pi' \varphi) \pi' + i\delta^{3}(\mathbf{x} - \mathbf{x}')\pi' = 2i\delta^{3}(\mathbf{x} - \mathbf{x}')\pi'## hence ##[\varphi,H] = i\int d^{3}x' \delta^{3}(\mathbf{x} - \mathbf{x}')\pi' = i\pi##. The Heisenberg equation of motion thus gives ##\dot{\varphi} = \pi##.

Similarly, ##2[\pi,H] = \int d^{3}x'([\pi,\pi'^2] + [\pi,(\nabla'\varphi')^2] + m^2[\pi,\varphi'^2])##; ##[\pi,\pi'] = 0## so ##[\pi,\pi'^2] = 0## and ##[\pi,\varphi'^2] = - 2i\delta^{3}(\mathbf{x} - \mathbf{x'})\varphi' ##.

Furthermore ## [\pi,(\nabla'\varphi')^2] = (\pi \nabla'\varphi')\cdot\nabla'\varphi' - \nabla'\varphi'\cdot((\nabla'\varphi')\pi) \\= (\nabla'\varphi')\cdot\pi (\nabla'\varphi') - i(\nabla'\delta^{3}(\mathbf{x} - \mathbf{x'}))\cdot\nabla'\varphi' - (\nabla'\varphi')\cdot\pi(\nabla'\varphi') - i\nabla'\varphi'\cdot(\nabla'\delta^{3}(\mathbf{x} - \mathbf{x'})) \\= -2i(\nabla'\delta^{3}(\mathbf{x} - \mathbf{x'}))\cdot\nabla'\varphi'##

This leaves us with ##[\pi,H] = -i\int d^{3}x'\delta^{3}(\mathbf{x} - \mathbf{x'})\varphi' - i\int d^{3}x'(\nabla'\delta^{3}(\mathbf{x} - \mathbf{x'}))\cdot\nabla'\varphi' = i(\nabla^2\varphi - m^2\varphi)##.

So now here's the part that I'm not sure about. The Heisenberg equation of motion derived above is valid for any scalar quantum field ##\varphi## right? In the derivation itself I think all I used was the fact that ##\varphi## is an arbitrary scalar field, making no reference to a specific scalar field. So it works for the field conjugate momentum ##\pi## as well? If so then we get ##i[\pi,H] = -\dot{\pi} = -\ddot{\varphi}## and at the same time ##i[\pi,H] = -(\nabla^2\varphi - m^2\varphi)## so that ##\ddot{\varphi} - \nabla^2\varphi + m^2\varphi = -\partial^{\mu}\partial_{\mu}\varphi+m^2\varphi = 0## as desired.

Does this all check out i.e. are all my calculations sound? I just want to make sure the calculations are sound so that I can move on to parts (d) and (e). Also if you know of a faster calculation to get the KG equation from the Heisenberg equation of motion then I'd appreciate it if you could show it. Thanks in advance!
 
  • Like
Likes 1 person
Physics news on Phys.org
  • #2
Actually part (d) was really short so I may as well post that too; hopefully it checks out :)

We have by definition ##P^{i} = -\int d^{3}x'(\pi' \partial'^{i}\varphi')## and from part (a) above we also have ##i[P^{\mu},\varphi] = -\partial^{\mu}\varphi##.

Now ##[\pi' \partial'^{i}\varphi',\varphi] \\= \pi'((\partial'^{i}\varphi') \varphi) - (\varphi\pi')\partial'^{i}\varphi' \\= \pi'\partial'^{i}(\varphi'\varphi)-\pi'(\varphi (\partial'^{i}\varphi')) - i\delta^{3}(\mathbf{x} - \mathbf{x'})\partial'^{i}\varphi' \\= \pi'\partial'^{i}[\varphi',\varphi] - i\delta^{3}(\mathbf{x} - \mathbf{x'})\partial'^{i}\varphi' = - i\delta^{3}(\mathbf{x} - \mathbf{x'})\partial'^{i}\varphi'##

hence ##i[P^{i},\varphi] = -i\int d^{3}x'[(\pi' \partial'^{i}\varphi'),\varphi] = -\int d^{3}x' \delta^{3}(\mathbf{x} - \mathbf{x'})\partial'^{i}\varphi' = -\partial^{i}\varphi## as desired.
 
  • #3
Also I'm stuck on part (e) so if someone could help me out with part (e) that would be swell :)

The free scalar field solution is ##\varphi(x) =\int d\tilde{k} [a(\mathbf{k})e^{i(\mathbf{k}\cdot \mathbf{x} - \omega t)} + a^{\dagger}(\mathbf{k})e^{i(-\mathbf{k}\cdot \mathbf{x} + \omega t)}]## where ##d\tilde{k} = \frac{d^3 k}{(2\pi)^3 2\omega}## is the Lorentz invariant measure.

Then ##\pi = \dot{\varphi} = \int d\tilde{k} [ -a(\mathbf{k})e^{i(\mathbf{k}\cdot \mathbf{x} - \omega t)} + a^{\dagger}(\mathbf{k})e^{i(-\mathbf{k}\cdot \mathbf{x} + \omega t)}]i\omega##

and ##\nabla \varphi = \int d\tilde{k} [a(\mathbf{k})e^{i(\mathbf{k}\cdot \mathbf{x} - \omega t)} - a^{\dagger}(\mathbf{k})e^{i(-\mathbf{k}\cdot \mathbf{x} + \omega t)}]i\mathbf{k}##

so we have ##\mathbf{P} = \int d\tilde{k}d\tilde{k}' d^{3}x[ -a(\mathbf{k})e^{i(\mathbf{k}\cdot \mathbf{x} - \omega t)} + a^{\dagger}(\mathbf{k})e^{i(-\mathbf{k}\cdot \mathbf{x} + \omega t)}] [a(\mathbf{k'})e^{i(\mathbf{k'}\cdot \mathbf{x} - \omega' t)} - a^{\dagger}(\mathbf{k'})e^{i(-\mathbf{k'}\cdot \mathbf{x} + \omega' t)}]\omega\mathbf{k'}\\ = \int d\tilde{k}d\tilde{k}' d^{3}x[-a(\mathbf{k})a(\mathbf{k'})e^{i(\mathbf{k}+\mathbf{k'})\cdot \mathbf{x}} e^{-i(\omega + \omega') t} + a^{\dagger}(\mathbf{k})a(\mathbf{k'})e^{i(\mathbf{k'}-\mathbf{k})\cdot \mathbf{x}} e^{-i(\omega - \omega') t} \\+a(\mathbf{k})a^{\dagger}(\mathbf{k'})e^{i(\mathbf{k}-\mathbf{k'})\cdot \mathbf{x}} e^{-i(\omega' - \omega) t} - a^{\dagger}(\mathbf{k})a^{\dagger}(\mathbf{k'})e^{-i(\mathbf{k}+\mathbf{k'})\cdot \mathbf{x}} e^{i(\omega + \omega') t}]\omega\mathbf{k'}\\ = -(2\pi)^3 \int d\tilde{k}d\tilde{k}'\delta^{3}(\mathbf{k} +\mathbf{k'})[a(\mathbf{k})a(\mathbf{k'})e^{-i(\omega + \omega') t}+ a^{\dagger}(\mathbf{k})a^{\dagger}(\mathbf{k'})e^{i(\omega + \omega') t}]\omega\mathbf{k'}\\+(2\pi)^3 \int d\tilde{k}d\tilde{k}'\delta^{3}(\mathbf{k} - \mathbf{k'})[a^{\dagger}(\mathbf{k})a(\mathbf{k'})e^{-i(\omega - \omega') t} +a(\mathbf{k})a^{\dagger}(\mathbf{k'})e^{-i(\omega' - \omega) t}]\omega\mathbf{k'}\\ = -\frac{1}{2} \int d\tilde{k}[a(\mathbf{k})a(\mathbf{-k})e^{-2i\omega t}+ a^{\dagger}(\mathbf{k})a^{\dagger}(\mathbf{-k})e^{2i\omega t}]\mathbf{k}\\+\frac{1}{2} \int d\tilde{k}[a^{\dagger}(\mathbf{k})a(\mathbf{k}) +a(\mathbf{k})a^{\dagger}(\mathbf{k})]\mathbf{k}##

I don't know how to simplify this any further. The second term in the final equality looks like it should stay intact since the Hamiltonian for the free scalar field is given by ##H = \frac{1}{2} \int d\tilde{k}[a^{\dagger}(\mathbf{k})a(\mathbf{k}) +a(\mathbf{k})a^{\dagger}(\mathbf{k})]\omega##.

I'm guessing it's the first term in the final equality that has to vanish somehow. However I can't seem to make it vanish. I tried using a parity inversion ##\mathbf{k}\rightarrow -\mathbf{k}## and the fact that ##[a(\mathbf{k}),a(\mathbf{-k})] = [a^{\dagger}(\mathbf{k}),a^{\dagger}(\mathbf{-k})] = 0## but the lone ##\mathbf{k}## in the integrand picks up an overall sign that cancels out with the overall sign picked up by the Lorentz invariant measure ##d\tilde{k}##, giving me back the original integral even though what I need is an overall negative of the original integral in order for it to vanish. Thanks in advance for any help.

EDIT: Oops nevermind! I forgot that the bounds of the integral also flip under the parity inversion so I have to flip them back and that gives me the overall negative sign that I need to make the first term vanish.
 
Last edited:
  • #4
Thanks wbn! :biggrin:
 
  • #5


Your calculations seem to be correct and thorough. The fact that you didn't make any specific reference to a particular scalar field in the derivation of the Heisenberg equation of motion is indeed correct, as it applies to any scalar field. Your conclusion that the Klein-Gordon equation is satisfied by the field is also correct.

For part (d), you can use the equal time commutation relation ##[\varphi(\mathbf{x}),\pi(\mathbf{y})] = i\delta^{3}(\mathbf{x} - \mathbf{y})## and the definition of the spatial momentum operator to show that ##[\mathbf{P},\varphi] = -\nabla\varphi##. Similarly, you can show that ##[\mathbf{P},\pi] = -\nabla\pi##. Then, using the Heisenberg equation of motion for both ##\varphi## and ##\pi##, you can show that ##[\mathbf{P},H] = i\nabla H##. This is the relation you derived in part (a).

For part (e), you can express ##\mathbf{P}## in terms of the creation and annihilation operators, using the Fourier transform of the field operator. Then, you can use the canonical commutation relations for the creation and annihilation operators to show that ##[\mathbf{P},a(\mathbf{k})] = -\mathbf{k}a(\mathbf{k})## and ##[\mathbf{P},a^{\dagger}(\mathbf{k})] = \mathbf{k}a^{\dagger}(\mathbf{k})##. This is the relation you were asked to derive in part (e).
 

FAQ: Srednicki Problem 3.4: Deriving the Klein-Gordon Equation

1. What is the Srednicki Problem 3.4?

The Srednicki Problem 3.4 is a physics problem proposed by Mark Srednicki in his textbook "Quantum Field Theory". It involves deriving the Klein-Gordon equation, which is a relativistic wave equation that describes the behavior of spinless particles.

2. Why is the Klein-Gordon equation important?

The Klein-Gordon equation is important because it is one of the fundamental equations in quantum field theory. It describes the behavior of spinless particles such as scalar bosons, and has applications in various fields of physics, including particle physics and cosmology.

3. What is the derivation process for the Klein-Gordon equation?

The derivation process for the Klein-Gordon equation involves starting with the relativistic energy-momentum relation and applying the principles of quantum mechanics. This results in a wave equation for a scalar field, which is then modified to include a mass term and the potential energy of the field.

4. What is the significance of the potential energy term in the Klein-Gordon equation?

The potential energy term in the Klein-Gordon equation accounts for the interactions between particles and their surroundings. It can describe the effects of external forces and can also be used to study the behavior of particles in a potential well.

5. How is the Klein-Gordon equation related to other equations in physics?

The Klein-Gordon equation is related to other equations in physics, such as the Schrödinger equation and the Dirac equation. It can be derived from the Schrödinger equation in the non-relativistic limit and can be extended to include spin in the form of the Dirac equation.

Back
Top